Sinusgetriebenes Zwei-Level-System (TLS)

Ich versuche, die Frage des angetriebenen Zwei-Ebenen-Systems (TLS oder Qubit) mithilfe einer Fourier-Transformation der Schrödinger-Gleichung (SHE) zu lösen, aber ich bleibe bei der Lösung der Gleichung hängen.

Gegeben Hamiltonian

H = ( ω 0 2 1 2 v e ich T ω D 1 2 v e ich T ω D ω 0 2 )

und Einstecken in SHE:

ich D dt ( C A ( T ) C B ( T ) ) = H ( C A ( T ) C B ( T ) )

Ich bekomme einen Satz von zwei gekoppelten Differentialgleichungen 1. Ordnung, die ich dann Fourier-transformiere und die Ableitungsregel und die Verschiebungsregel verwende, um zu erhalten:

\links(

ω C A ( ω ) = 1 2 v C B ( ω ω D ) 1 2 ω 0 C A ( ω ) ω C B ( ω ) = 1 2 v C A ( ω + ω D ) + 1 2 ω 0 C B ( ω )
\Rechts)

Wenn ich die verschiebe C B Gleichung:

ω C B ( ω ω D ) = 1 2 v C A ( ω D ω D + ω ) + + 1 2 ω 0 C B ( ω ω D )

und nach Term auflösen, den ich wieder in die erste Gleichung einsetzen kann:

C B ( ω ω D ) = v C A ( ω ) 2 ω + ω 0

und löse nach C A :

C A ( ω ) = v 2 C A ( ω ) ( 2 ω ) 2 + ω 0 2

Es sieht also aus wie eine Lorentz-Funktion mit einer Breite der Resonanzfrequenz und zentriert bei 0 multipliziert C A , Aber C A bricht ab, es sei denn, es ist 0? oder eine Delta-Funktion?

Hier verstehe ich nicht, wie ich vorgehen soll, um das Problem weiter zu lösen? Ist Teil der Herausforderung, dass ich das Problem ohne Randbedingungen löse, nur versuche, den stationären Zustand zu finden?

Vielleicht finden Sie meine Antwort auf diese Frage ( physics.stackexchange.com/questions/138765/… ) nützlich.

Antworten (1)

Die Lösung besteht darin, zu erkennen, dass die stationäre Lösung eines harmonisch angetriebenen Systems auch harmonisch schwingen muss. (In Bezug auf Ihre Lösung bedeutet dies, dass spektral die C ich ( ω ) sind Deltafunktionen, die Ihren Widerspruch auflösen.)

Daher beginnt man üblicherweise damit, den oszillatorischen Ansatz zu postulieren

C A ( T ) = C A e ich ω A T , C B ( T ) = C B e ich ω B T ,
bei dem die C A Und C B sind jetzt Konstanten. Als Ansatz ist dies harmlos und wenn es sich herausstellt, dass es keine Lösung ist, können Sie es fallen lassen (aber es wird passieren). Ihre Schrödinger-Gleichung lautet also
( ω 0 2 1 2 v e ich T ω D 1 2 v e ich T ω D ω 0 2 ) ( C A ( T ) C B ( T ) ) = ich D D T ( C A ( T ) C B ( T ) )
So
( ω 0 2 1 2 v e ich T ω D 1 2 v e ich T ω D ω 0 2 ) ( C A e ich ω A T C B e ich ω B T ) = ( ω A C A e ich ω A T ω B C B e ich ω B T )
oder
{ ω 0 2 C A e ich ω A T + 1 2 v e ich T ω D C B e ich ω B T = ω A C A e ich ω A T , 1 2 v e ich T ω D C A e ich ω A T + ω 0 2 C B e ich ω B T = ω B C B e ich ω B T .
Dies müssen Sie einstellen ω A + ω D = ω B , danach können Sie die Zeitabhängigkeit eliminieren. Damit bleibt Ihnen das einfache lineare System
{ ω 0 2 C A + 1 2 v C B = ω A C A , + 1 2 v C A + ω 0 2 C B = ( ω A + ω D ) C B ,
was ein Eigenwertsystem für den Hamiltonian ist H = ( ω 0 2 1 2 v 1 2 v ω 0 2 ω D ) .

Da Sie dies als markiert haben, lasse ich die Berechnung hier, da Sie sicher besser dran sind, Eigenvektoren und Eigenwerte selbst zu berechnen.

Entschuldigung, warum ist ω A + ω D = ω B ? und welche Delta-Funktion ist eine Lösung für meine obige Gleichung? δ ( ω ) oder δ ( ω ω D ) oder δ ( ω ω 0 ) ?
@AimForClarity Sie möchten, dass die letzte Gleichung mit Exponentialen für alle Zeiten gilt, was bedeutet, dass alle Exponentiale identisch sein müssen, sodass ihre Frequenzen übereinstimmen müssen.
Ich würde Sie davon abhalten, in dieser Fourier-Darstellung zu arbeiten (es verschleiert nur die Tatsache, dass die Komponenten einfach oszillierende Exponentiale sind). Wenn Sie das wissen müssen, dann lösen Sie das letzte Eigensystem nach auf ω A Und ω B , und Fourier-Transformation des anfänglichen Ansatzes für C A ( T ) Und C B ( T ) .
Vergessen Sie übrigens nicht, dass Sie Antworten positiv bewerten und akzeptieren können , die Sie für nützlich hielten!
Jetzt, wo ich darüber nachdenke, kann ich abbrechen C A In C A ( ω ) = v 2 C A ( ω ) ( 2 ω ) 2 + ω 0 2 und löse nach Omega auf, was mir genau die richtige Rabi-Frequenz gibt: Ω = ( v 2 ) 2 ( ω 0 2 ) 2 . Dies sagt mir, dass die Gleichung für alle Nicht-0 oder Nicht-Unendlichen gelten wird C A Und C B nur bei ω = ± Ω , und überall sonst C A Und C B sollte 0 sein (oder Unendlich, aber das lassen wir als unphysikalisch weg).
Mit anderen Worten, die Lösung ist eine Delta-Funktion bei ω = oder ± Ω C A = C A ' δ ( ω Ω )  +  C A δ ( ω Ω ) , Wo C A , C B sind beliebige C-Nummern.
@AimForClarity Ihr letzter Kommentar wurde komplett verstümmelt.
Entschuldigung, ich war noch am Tippen, jetzt sollte alles da sein
Ja, im Fourier-Raum ist es eine Delta-Funktion. Aber noch einmal, es ist nichts zu gewinnen, wenn man nicht im Zeitbereich arbeitet.